Almost 60 followers problem

Algebra Level 5

10 x 2 + 10 y 2 + z 2 \large 10x^2+10y^2+z^2 Let x , y , z x,y,z be positive real numbers that satisfy x y + y z + x z = 1 xy+yz+xz=1 .

Find the minimum value of the expression above.

Give your answer to 2 decimal places


The answer is 4.00.

This section requires Javascript.
You are seeing this because something didn't load right. We suggest you, (a) try refreshing the page, (b) enabling javascript if it is disabled on your browser and, finally, (c) loading the non-javascript version of this page . We're sorry about the hassle.

5 solutions

Otto Bretscher
Apr 30, 2016

Happy Victory Day, Comrade!

( 2 x 2 y ) 2 + ( 4 y z ) 2 + ( 4 x z ) 2 = 20 x 2 + 20 y 2 + 2 z 2 8 x y 8 y z 8 x z 0 (2x-2y)^2+(4y-z)^2+(4x-z)^2=20x^2+20y^2+2z^2-8xy-8yz-8xz\geq 0 so that 10 x 2 + 10 y 2 + z 2 4 x y + 4 y z + 4 x z = 4 10x^2+10y^2+z^2\geq 4xy+4yz+4xz=4 . The minimum is 4 \boxed{4} , attained when x = y = 1 3 x=y=\frac{1}{3} and z = 4 3 z=\frac{4}{3}

Nice solution from the Master again. Must really relearn eigenvalues. Upvote!

Chew-Seong Cheong - 5 years, 1 month ago

How did you group them so nicely in the first place? Some eigenvalues + kernel thing?

Pi Han Goh - 5 years, 1 month ago

Log in to reply

Yes, exactly; you can read my thoughts now (a little scary...). I used eigenvectors to find that z = 4 x = 4 y z=4x=4y ... then we know how to group them ;) Writing it up is much quicker if we just form squares.

Otto Bretscher - 5 years, 1 month ago

Log in to reply

Be right back. Doing some fancy linear algebra.

Pi Han Goh - 5 years, 1 month ago

Log in to reply

@Pi Han Goh I'm doing some (not so fancy) house cleaning... I will not be online much today.

Otto Bretscher - 5 years, 1 month ago

Log in to reply

@Otto Bretscher Okay I got the eigenvalues for the quadratic form for (10x^2+10y^2 + z^2) to be 1, 10, 10. What's next?

Pi Han Goh - 5 years, 1 month ago

@Otto Bretscher Can you explain the methos / give some reference please? Thank you!

Noam Pirani - 5 years, 1 month ago
Manuel Kahayon
May 3, 2016

AM-GM? I used Cauchy-schwarz TT.TT

( 1 2 + 1 2 + 2 2 ) ( ( 2 x ) 2 + ( 2 y ) 2 + z 2 ) ( 2 x + 2 y + 2 z ) 2 (1^2+1^2+2^2)((2x)^2+(2y)^2+z^2) \geq (2x+2y+2z)^2

( 6 ) ( 4 x 2 + 4 y 2 + z 2 ) 4 ( x + y + z ) 2 (6)(4x^2+4y^2+z^2) \geq 4(x+y+z)^2

4 x 2 + 4 y 2 + z 2 2 3 ( x 2 + y 2 + z 2 + 2 ( x y + y z + x z ) ) 4x^2+4y^2+z^2 \geq \frac{2}{3} (x^2+y^2+z^2+2(xy+yz+xz))

4 x 2 + 4 y 2 + z 2 2 3 ( x 2 + y 2 + z 2 + 2 ) 4x^2+4y^2+z^2 \geq \frac{2}{3} (x^2+y^2+z^2+2) (Since x y + y z + x z = 1 xy+yz+xz=1 )

10 x 2 + 10 y 2 + z 2 3 4 3 \frac{10x^2+10y^2+z^2}{3} \geq \frac{4}{3} (Transposition)

10 x 2 + 10 y 2 + z 2 4 10x^2+10y^2+z^2 \geq \boxed {4}

great solution, an upvote for you my friend, but it supposed to be 1 2 + 1 2 + 2 2 1^2+1^2+2^2 on the LHS

P C - 5 years, 1 month ago

Log in to reply

Whoah, sorry, didn't notice that... Thanks anyway :)

Manuel Kahayon - 5 years, 1 month ago
Anthony Ritz
May 1, 2016

I first tried symmetry in x x , y y , and z z , but it didn't work -- presumably because the two equations are not mutually symmetric in z z (substituting just y = x y=x gives the expression as 20 x 2 + z 2 20x^2+z^2 and the equation as x 2 + 2 x z = 1 x^2+2xz=1 ). But if you want to go this way, you could take just the symmetry in x x and y y and proceed from there:

Solve the previous equation for z z to get z = 1 x 2 2 x z=\dfrac{1-x^2}{2x} and z 2 = 1 2 x 2 + x 4 4 x 2 = 1 4 x 2 1 2 + 1 4 x 2 z^2=\dfrac{1-2x^2+x^4}{4x^2}=\frac{1}{4}x^{-2}-\frac{1}{2}+\frac{1}{4}x^2 .

Substitute into the expression to get 20 x 2 + 1 4 x 2 1 2 + 1 4 x 2 = 1 4 x 2 1 2 + 81 4 x 2 20x^2+\frac{1}{4}x^{-2}-\frac{1}{2}+\frac{1}{4}x^2=\frac{1}{4}x^{-2}-\frac{1}{2}+\frac{81}{4}x^2 .

Now, by AM-GM, 1 4 x 2 1 2 + 81 4 x 2 1 2 + 2 1 4 x 2 81 4 x 2 = 1 2 + 2 ( 9 4 ) = 4 \frac{1}{4}x^{-2}-\frac{1}{2}+\frac{81}{4}x^2\geq-\frac{1}{2}+2\sqrt{\frac{1}{4}x^{-2}*\frac{81}{4}x^2}=-\frac{1}{2}+2(\frac{9}{4})=\boxed{4} .

Equality occurs when 81 4 x 2 = 1 4 x 2 \frac{81}{4}x^2=\frac{1}{4}x^{-2} , i.e. x 4 = 1 81 x^4=\frac{1}{81} , i.e. x = 1 3 x=\frac{1}{3} , y = 1 3 y=\frac{1}{3} , z = 4 3 z=\frac{4}{3} .

Aakash Khandelwal
Apr 30, 2016

Let x = κ , y = κ , z = 10 κ x=\kappa , y=\kappa , z= \sqrt{10}\kappa .

The above ratio is taken so as to have symmetry to get minimum.

Thus κ = 1 / ( 1 + 2 10 ) \kappa= 1/(1+2\sqrt{10}) .

Now putting in given expression min value is

4.095 \approx 4.095 .

No, it's exactly 4 ;)

Otto Bretscher - 5 years, 1 month ago

0 pending reports

×

Problem Loading...

Note Loading...

Set Loading...